How to find the volume of a square with function-based side?

  • Thread starter Thread starter Eclair_de_XII
  • Start date Start date
  • Tags Tags
    Square Volume
Click For Summary
To find the volume of the solid with a base bounded by the curves y = x² and y = 2 - x², the correct area function for the square cross-sections is A(y) = (2 - 2x²)², leading to the integral V = ∫A(f(x))dx. The initial calculations contained errors, particularly in the area function and volume integral setup. After correcting these mistakes, the volume was recalculated to be V = 64/15, matching the answer in the back of the book. Proper setup and careful evaluation of the integral are crucial for obtaining the correct volume.
Eclair_de_XII
Messages
1,082
Reaction score
91

Homework Statement


"[Find the volume of a] solid whose base is the region bounded by the curves (y = x2) and y = 2 - x2 and whose cross sections through the solid perpendicular to the x-axis are squares."

Homework Equations


A(f(x)) = f(x)2
V = ∫(A(f(x))dx
Image of problem: http://i.imgur.com/6FtsBzO.png
Answer (from back of book): V = 64/15.


The Attempt at a Solution


y = (2 - x2) - (x2) = 2 - 2x2
A(y) = (2 - 2x2)2 = 4x4 - 4x2 + 4
V = ∫A(f(x)) = (4/5)x5 - (4/3)x3 + 4x on [-1,1]
[(4/5) - (4/3) + 4] = [(12 - 20 + 60)/(15)] - [(-12 + 20 - 60)/(15)] = (104/15) ≠ (64/15)

I think I'm doing this right, but I'm not getting the right numbers. Could someone help me with this?
 
Physics news on Phys.org
Eclair_de_XII said:

Homework Statement


"[Find the volume of a] solid whose base is the region bounded by the curves (y = x2) and y = 2 - x2 and whose cross sections through the solid perpendicular to the x-axis are squares."

Homework Equations


A(f(x)) = f(x)2
V = ∫(A(f(x))dx
Image of problem: http://i.imgur.com/6FtsBzO.png
Answer (from back of book): V = 64/15.


The Attempt at a Solution


y = (2 - x2) - (x2) = 2 - 2x2
A(y) = (2 - 2x2)2 = 4x4 - 4x2 + 4
V = ∫A(f(x)) = (4/5)x5 - (4/3)x3 + 4x on [-1,1]
[(4/5) - (4/3) + 4] = [(12 - 20 + 60)/(15)] - [(-12 + 20 - 60)/(15)] = (104/15) ≠ (64/15)

I think I'm doing this right, but I'm not getting the right numbers. Could someone help me with this?

Check your setup math, (x^2-(2-x^2))^2 would be the region you want to look at.

You have two fours, I think you just made a math mistake. =)
 
You're right.

A(y) = (2 - 2x2)2 = 4x4 - 8x2 + 4
V = ∫A(f(x)) = (4/5)x5 - (8/3)x3 + 4x on [-1,1]
[(4/5) - (8/3) + (4)] - [(-4/5) + (8/3) - (4)] = 2(12 - 40 + 60)/(15) = 2(32/15) = 64/15

Thanks!
 
  • Like
Likes Student100
Question: A clock's minute hand has length 4 and its hour hand has length 3. What is the distance between the tips at the moment when it is increasing most rapidly?(Putnam Exam Question) Answer: Making assumption that both the hands moves at constant angular velocities, the answer is ## \sqrt{7} .## But don't you think this assumption is somewhat doubtful and wrong?

Similar threads

Replies
2
Views
1K
Replies
12
Views
2K
  • · Replies 4 ·
Replies
4
Views
1K
Replies
2
Views
2K
  • · Replies 3 ·
Replies
3
Views
2K
  • · Replies 6 ·
Replies
6
Views
3K
  • · Replies 8 ·
Replies
8
Views
2K
Replies
1
Views
1K
  • · Replies 2 ·
Replies
2
Views
2K
Replies
3
Views
2K